1. (C). You are asked to find a conclusion that the given information supports. For this sort of problem, always stick
as close as you can to the passage. The guidelines call for including vegetables (not fruits) in every meal. The school
board has replaced fries with fruit. While this does sound like a nutritional improvement, all you can infer for certain
is that the guidelines are not being met.
(A) You know nothing from the passage about the relative health value of fruits and vegetables.
(B) Likewise, the passage provides no information about how likely students are to eat any kind of food. Avoid
bringing in outside knowledge or suppositions, when you are trying to draw a proper conclusion from given
information.
(C) CORRECT. This choice spells out the only inference you can legally make: the board is not following the
guidelines.
(D) You do not know whether the board is responsible for the health of the students. It may be, or it may not be.
(E) This opinion may seem reasonable, but it is not at all proven by the passage, which simply presents facts. In
general, avoid making value judgments when drawing a conclusion from given factual information. The premises would
have to contain opinions as well.
2. (C). You are asked to strengthen the conclusion, which is public officials shouldn't buy lottery tickets (as
indicated by Therefore). Some of the answer choices provide reasons why no one should buy lottery tickets.
However, you are looking for a choice that supports this specific conclusion in the context of this argument by linking
the key premise (buying lottery tickets is a form of gambling) to the conclusion. You need either to state the missing
assumption or provide direct support for it.
(A) People who play the lottery are not likely to win, it's true. This is a reason why people in general should not buy
lottery tickets. But you need a stronger link between premise and conclusion.
(B) It's irrelevant whether some public officials are guilty of more serious offenses than gambling.
(C) CORRECT. This choice provides the link you seek. Premise: Buying lottery tickets = gambling. Conclusion:
Public officials shouldn't buy lottery tickets. The missing assumption, which this choice supplies word for word, is
Public officials shouldn't gamble. If an argument says Certain people shouldn't do X because X is Y, then you need
to assume that those people shouldn't do Y.
(D) It may be true that many officials are tempted to violate the rules, but even if so, it wouldn't strengthen a moral
prohibition (the conclusion tells you what public officials shouldn't do).
(E) Like choice (A), this choice provides a reason why people in general shouldn't play the lottery, but it does not
provide as strong support for the conclusion as choice (C) does
.
3. (C). The question asks you to support a judgment (that is, a conclusion) on the basis of the statements in the
passage. Remember to stay close to those statements— do not make additional assumptions. Look for what you can
prove.
(A) Careful! You don't know whether Tunbridge College provides the best value in the state — the speaker has
simply pointed out that Saddlebrook can't be the best value, since another college is a better value (in a variety of
ways). It could be that Saddlebrook and Tunbridge are actually the 10th and 7th best values in the state, for instance.
(Think about when you say things like, "Uncle Jay thinks he's the smartest person in our family? Even my eight-yearold
is better at math than he is!" You're probably not arguing that your eight-year-old is the smartest person in the
family — you're just pointing out that Uncle Jay can't be the best, since at least one other person is above him.)
(B) This choice is not a judgment that could be proven from premises. If you added it to the premises, it would
perhaps weaken the argument that Tunbridge is a better value than Saddlebrook (for instance, higher-caliber students
might be more likely to earn larger incomes after graduation, regardless of the value of the education).
(C) CORRECT. This choice takes a much safer position than choice (A). The second sentence in the passage
describes various ways in which Tunbridge is a better value than Saddlebrook, and so you can conclude that
Saddlebrook is not the best value.
(D) This choice represents an assumption in the argument — that income after graduation is part of evaluating which
college provides the best value — but you are being asked here for a conclusion, not an assumption within the
argument.
(E) This choice is irrelevant — satisfaction is not the same as value.
4. (B). This is a standard argument where the author (or, in this case, the researchers) have confused correlation with
causation. If two traits X and Y are found together (in this case, red cars and speeding tickets), you do not have any
particular reason to believe that X causes Y. After all, it could be that Y causes X, or that some third factor Z causes
both (in this case, the type of person who buys a red car also tends to get speeding tickets). In addition, you should
notice that the correlation is specifically observed between the color of the car and the number of speeding tickets.
As most every driver knows, getting a speeding ticket depends on more than just speeding (driver behavior) — you
also have to be unlucky enough to catch the attention of a police officer. Thus, the researchers have built in a second
assumption — that the increased number of speeding tickets is the result of an increase in the number of speeding
infractions (and not, say, the increase in attention that a red car might get).
(A) This choice regarding black cars is completely irrelevant to this argument.
(B) CORRECT. If the researchers believe this, they've eliminated the idea that the reverse causation could be
possible. That is, this statement gets rid of the possibility that the color, rather than the increased speeding, is what is
responsible for the increase in tickets. Note that this assumption is not the only one the argument requires, but it is
certainly required by the argument.
(C) What color cars police officers drive does not matter at all.
(D) This choice focuses on an entirely different type of ticket — a parking ticket — and is thus irrelevant. You have
no information regarding tendency to speed, which is necessary for the researcher's argument.
(E) This choice is outside the scope of the argument, as it deals with events after the act of speeding or the issuance of
the ticket. It may give hope to drivers of red cars, but is completely irrelevant to the question at hand.
5. (E). The headmaster's argument explicitly relies on the idea that students perform better on math tests when they
are fully awake. Noting the drop in scores, the headmaster reasons that this drop is due to the change in class time as
opposed to any other factor or combination of factors. The assumption you are looking for must connect the dots
between the time of the class and the waking state of the students.
(A) The logistics of the schedule are irrelevant to the headmaster's position that scores would improve if the class
were moved.
(B) The time at which other schools hold math classes is irrelevant to the headmaster's argument, especially since the
efficacy of that scheduling is not mentioned.
(C) The idea that the quality of teaching has little bearing on scores (whether or not you agree with this in the real
world!) might provide a tiny bit of support to the headmaster's idea. After all, if the quality of teaching hardly
influences test scores, another factor might be expected to have greater impact — and that factor might be time of
day. However, it is a huge stretch to say that one must assume that some other factor (such as teaching quality) has
little effect on test scores, in order to conclude that time of day could influence those scores.
(D) provides an alternative explanation as to why the scores might have dropped this year (the use of an unfamiliar
curriculum), but this explanation is not an assumption upon which the headmaster's argument depends.
(E) CORRECT. This choice indicates the direct link needed between time of day and student wakefulness.
6. (E). ITEL believes that it hasn't lost its valuable employees after layoffs because it has a highly risk-tolerant
environment. To further support this claim, you need information that supports the idea that this specific reason is
why ITEL hasn't lost any of its valuable employees after the previous four layoffs.
(A) This choice offers one reason why those who do change their jobs might do so, but the conclusion in question
here is why certain people would keep their jobs, not change their jobs.
(B) This is almost certainly true in the real world, but it does not directly address the specific question here: why
ITEL claims it is able to retain its most valuable employees. Even if you say that this choice does offer an alternate
explanation for the retention of employees (if a company handles the layoffs well, valuable employees may be more
likely to stay), then this choice would actually weaken the conclusion. It would offer an alternate explanation as to why
ITEL is retaining its employees, not the reason ITEL claims.
(C) This choice does the opposite of what you want: it weakens the conclusion. If people are staying with ITEL
because they cannot find a new job, then ITEL is not correct in claiming that people are choosing to stay because of
the highly risk-tolerant environment.
(D) Like choice (B), this choice is very likely true in the real world, but it does not address the specific question here:
why ITEL claims it is able to retain its most valuable employees. As such, it cannot strengthen ITEL's claim that its
high risk tolerance allows it to retain its employees.
(E) CORRECT. If intolerance toward risk is considered a significant detriment by employees in the industry, then
having a higher risk-tolerance environment would be considered an advantage. This strengthens ITEL's claim that its
high risk-tolerance helps it to retain employees.
7. (C). The exterminator explains the effects of using poisoned food to combat a mouse infestation: mice will pick up
the food and carry it to the nest, where all of the mice will eat the food and die as a result. The exterminator then
claims that if X happens, then it will be certain that Y happened first. Specifically, he claims that if all signs of the
mice disappear for three consecutive weeks after setting out poisoned food, then the mice all must have died as a
result of that poisoned food. The correct answer will weaken this claim, for instance by showing that if X happens, it
will not necessarily mean that Y happened; perhaps there is another reason why X happened.
(A) This choice is very tempting, but the exterminator argues that "all signs" of the mice "disappear"; this choice
mentions only the difficulty of seeing and hearing mice. These do not represent "all signs" of an infestation; mice may
leave droppings, chew things up, and so on. Further, the mice are not harder to see and hear only after poison has been
used. They are always hard to see and hear, so this does not provide an alternate explanation as to why signs might
diminish or disappear after poison has been used.
(B) This choice may be true, but it does not address the exterminator's claim that if all signs disappear, then the
poison must have killed the mice.
(C) CORRECT. This choice offers an alternate explanation for the absence of signs of an infestation. If the mice
have left the house, then any signs of infestation would also cease because the mice are no longer there. However, the
signs will have ceased because the mice left voluntarily, not because they have been eradicated by the poison, as
claimed by the exterminator.
(D) It is likely true that different poisons have different levels of efficacy, but this argument does not distinguish
between types of poison. The claim is simply that the poison must have killed the mice (as opposed to some other
explanation for why the signs of mice would disappear).
(E) This is another tempting choice; however, the exterminator does not claim that the poison will work within three
weeks (or within any time frame at all). He only claims that if the signs disappear over three consecutive weeks, then
that means that the poison worked. It might be the case that the signs don't disappear until weeks 3, 4, and 5, but the
exterminator's claim would still hold: because the signs have disappeared for three consecutive weeks, the poison
worked.
8. (E). The question presents seemingly contradictory premises. On the one hand, ammonium triiodide is a powerful
explosive that is easy to make from just two ingredients. On the other hand, apparently no terrorists have used this
chemical in attacks. The correct answer will reconcile these premises by presenting a relevant, reasonable explanation
for why terrorists have not used this explosive, despite its advantages. In other words, there must be a clear and serious
obstacle preventing the use of this chemical.
(A) This answer choice does present an obstacle: one of the ingredients has limited availability. However, the obstacle
is not described as serious. The ingredient can only be obtained from somewhat more restricted sources than a
grocery store. For example, chemical supply houses can provide the ingredient.
(B) Other explosives can be made from ammonia, but this fact does not satisfactorily explain why terrorists have not
used this explosive.
(C) This answer choice presents another seeming obstacle: many terrorists are more focused on weapons of mass
destruction than on these kinds of chemical explosives. However, this does not mean that all terrorists are more
focused on weapons of mass destruction. Moreover, even terrorists who are more focused on more devastating
weapons may still be interested enough in conventional explosives to use ammonium triiodide in some attack.
(D) Airport security devices may be calibrated to detect ammonium compounds, an obstacle to their use in attacks on
planes or airports. But terrorists might still use such compounds in other sorts of attacks.
(E) CORRECT. Only this answer choice presents a clear and serious obstacle to the use of ammonium triiodide in
terrorist attacks. Since the chemical is unstable when dry, terrorists are unlikely to use the chemical in attacks, since
some part of it might dry out and cause an early explosion.
9. (C). The conclusion is that "if major industries increase their capital reserves, the employment rate will not decline
in the future." Why? Because major industry did not have capital reserves. The author assumes that having capital
reserves is sufficient to prevent a decline in the employment rate. You are asked to cast doubt on the author's claim.
(A) Whether the drop in employment was foreseen does not relate to the core of the argument, which is that capital
reserves will prevent another decline in the employment rate.
(B) The fact that some major industries had appreciable capital reserves does not contradict the claim that an increase
in these reserves would prevent a future drop in employment rates.
(C) CORRECT. The author neglects to take into account the fact that other factors, such as an increase in labor costs,
could adversely affect the employment rate. For example, if the cost of labor becomes prohibitively expensive, even
with increased reserves, the employment rate could decline.
(D) Legislation mandating a certain level of reserves does not contradict the claim that increased reserves would
prevent a drop in employment rates.
(E) The fact that the employment rate was more severe this year than last does not contradict the claim that an
increase in reserves would prevent a drop in the employment rate.
10. (E). The conclusion of the argument is that new methods of math education in this country do a disservice to
our children. Why? Because math teachers emphasize in higher grades the same narrow, skills-based approach
that students learned in lower grades rather than the analytical tools they will need to solve complex math
problems. In order to weaken the conclusion, you need to show that this approach has not had a negative effect on
children's math skills.
(A) Music is used simply as an analogy in the argument. It is not central to the logic.
(B) The argument suggests that students are getting worse at advanced math skills due to new methods of teaching.
This answer choice provides no basis for evaluating students' math performance over time; you do not know how
performing "no worse than" students from other countries compares with students' performance prior to the
introduction of the new teaching methods.
(C) The simple fact that older students arrive at different answers does not address the concerns of the argument: that
students are not prepared for higher-level math.
(D) The fact that older students receive better grades in math does not address the concerns of the argument: that
students are not prepared for higher-level math.
(E) CORRECT. This choice states that an increasing percentage of native first-year students qualify to take college
courses requiring advanced math. This would seem to suggest that more children are prepared for advanced math than
had previously been the case, thus weakening the conclusion of the argument
.
11. (B). The first boldface sentence states that the fight against the drug trade in Country X should focus for the time
being on tightening the country's borders and targeting its major smugglers. The second boldface sentence states that
the United Nations and the government of Country X should eventually replace the poppy fields with other farming
ventures ("agricultural infrastructure"). You need to find a choice that describes the relationship between these two
sentences.
(A) This choice states that the first sentence is the conclusion and that the second sentence is an alternative to that
conclusion. This misrepresents the relationship. The first sentence is a shorter-term conclusion, and the second
sentence is a longer-term conclusion of the argument.
(B) CORRECT. This choice states that the first sentence is a short-term solution to a problem and the second a longterm
solution to the same problem. This accurately describes the relationship. The first sentence states that the fight
should focus for the time being on borders and smuggling while the second sentence states that the United Nations
and the government of Country X should work to replace the poppy crop with something else.
(C) This choice states that the first sentence presents a problem. According to the text, however, the first sentence is
not a problem but a solution to a problem.
(D) This choice states that the first sentence presents a popular solution to a problem. Since you have no way of
knowing whether the solution is popular, this cannot be correct.
(E) This choice states that the first sentence presents an argument, and that the second sentence provides evidence to
support that argument. Though the first sentence does present an argument, the second sentence, rather than providing
evidence, presents a second argument.
12. (B). The conclusion is that a developer who wishes to make a large profit would be wise to buy urban waterfront
lots and erect residential buildings on them. The basis for that claim is that people pay large sums for beach front
homes. You are asked to strengthen this argument.
(A) This choice states that people have more buying power today than in previous centuries. This does not strengthen
the claim that a developer will make money on urban waterfront properties.
(B) CORRECT. This choice states that homeowners will be willing to spend large sums of money on residential
properties in traditionally industrial or commercial districts. Since you know from the argument that urban waterfronts
have traditionally been industrial, this fact strengthens the claim that a developer can make a profit on urban
waterfront properties.
(C) This choice states that many urban waterfront lots are available for purchase. This does not suggest, however, that
a developer will be able to sell them after he or she builds on them.
(D) This choice states that many coastal cities are giving tax breaks to developers who rehabilitate the waterfront, but
this does not suggest that anyone will buy the developed properties.
(E) This choice states that properties in the interior of cities are more expensive than those on the waterfront.
Although waterfront properties are therefore cheaper to acquire, this does not necessarily mean that a developer can
make a profit after buying such properties.
13. (D). The passage states that mentally ill people fare best when two conditions are met: (1) they are not confined to
institutions, and (2) they receive high-quality outpatient care. When Congress allowed many mentally ill people to
leave mental institutions, it ensured that condition (1) was met for those who were released. The passage does not tell
you, however, whether Congress also ensured that condition (2) would be met. The plight of the mentally ill in 1983
would not be very surprising if those who had been released never received the high-quality outpatient care that they
needed.
(A) An increase in the number of people diagnosed with psychiatric disorders does not explain why the average
mentally ill person would be faring worse.
(B) The difference between outcomes for men and women is interesting, but without making additional assumptions,
you cannot use this difference to explain the drop in average welfare for the mentally ill.
(C) The discovery of new medications does not explain why the mentally ill were doing worse in 1983; if anything, it
makes their plight somewhat more surprising, since the new medications may have benefited some people with mental
illness.
(D) CORRECT. This resolves the paradox by offering a plausible explanation of why the mentally ill were worse off
in 1983. If Congress never supplied the funding that would have been necessary to provide high-quality outpatient care
to the newly-released patients, those patients did not fulfill condition (2) for improved welfare in psychiatric patients.
(E) The specific diagnoses of those who fared worst in 1983 cannot resolve the paradox under consideration, since
those individuals may well have had the same illnesses before they were released.
14. (A). The official's conclusion is that people who claim that the US is more vulnerable than other nations because
of the country's lack of a national vaccine laboratory are disloyal and incorrect. His basis for that conclusion is that
the US has generally long life span and low infant mortality relative to all UN countries. Moreover, he cites the high
quality of American hospitals, and he adds that many people he knows from around the world come to the US for
medical care. You are asked to find the choice that does NOT point out a weakness or potential weakness in the
official's argument.
(A) CORRECT. This choice states that the high quality of hospitals is not a factor affecting the public's vulnerability
to infectious disease. However, the quality of hospitals very arguably does affect how vulnerable the public is to such
disease.
(B) This choice highlights the official's logical jump from "disloyal" to "wrong" in the phrase these critics are
disloyal and thus wrong about the public's vulnerability. There is no necessary connection between disloyalty and
wrongness.
(C) If the Europeans that the official cited overwhelmingly consisted of wealthy men over the age of fifty, then the
official relied on an unrepresentative sample to justify his claim. What is true of wealthy older European men is not
necessarily true of Europeans or non-Americans generally.
(D) If the average life span of Americans is determined by causes other than infectious disease, then the official is not
limiting his evidence to cases relating to vaccines. Other causes of death are not relevant to the critics' argument.
(E) The ranking of the United States relative to all UN countries is misleading, since it does not compare the US to
other "advanced industrialized" or "developed" nations, as the critics' claim does.
15. (C). The conclusion of the argument is that renewable sources of energy, chiefly solar and wind, will be less risky
for certain utilities than nonrenewable sources, such as oil and gas. The basis for this claim is that the renewable
sources will provide stable, low-cost supplies of energy, whereas the prices for nonrenewable sources will fluctuate
according to availability. You are asked to find an assumption underlying this argument. In order for this argument to
be valid, it must in fact be true that these renewable sources of energy will provide stable, low-cost supplies.
(A) The utility companies' claim has to do with the supply risk of the new energy sources, not with how these sources
are received by the public.
(B) If no new supplies of traditional energy sources are found, then it is true that perhaps these nonrenewable supplies
will continue to fluctuate in price in a risky manner. However, the argument does not depend upon any assumption
about the future discovery of oil and gas supplies.
(C) CORRECT. If you assume that weather patterns are consistent and predictable, then with the stated premises, you
can conclude that solar and wind power will be less risky than oil and gas. If, on the other hand, weather patterns are
not consistent and predictable, then solar and wind power are not reliable and thus will not provide stable energy
supplies at low cost. Thus, the argument's conclusion directly depends on this assumption.
(D) To reach the required conclusion, it is not necessary to assume that the conversion technology for new sources is
not more expensive than the present technology.
(E) This choice does not directly affect the argument. Whether or not energy produced through combustion can be
made less risky, the new energy sources might still be less risky than the older sources.
16. (D). The conclusion of the argument is that one need not worry about ingesting pesticides when purchasing
produce from farms that use only organic pesticides. The basis for that claim is the fact that organic pesticides leave
the surface of produce within a few hours of spraying. In order for this argument to be valid, you must assume that the
organic pesticides do not harm the produce in any lasting way.
(A) This is unrelated to the argument, since the conclusion speaks about not having to worry about ingesting produce
on which only organic pesticides were known to be used.
(B) If anything, this statement runs counter to what the argument is saying. If produce that has been sprayed with
organic pesticide reaches the final consumer within hours after it is picked, it is possible that the consumer does need
be concerned about the pesticides.
(C) The conclusion of the argument is already limited to those farms which use "only organic pesticides."
(D) CORRECT. If a pesticide is capable of penetrating the skin of a fruit or vegetable then, while the organic
pesticide will dissipate from the surface of the fruit in a few hours, it may remain inside the fruit. The author of this
argument assumes that the pesticides cannot penetrate the skin.
(E) The issue of cost is unrelated to the argument's conclusion about pesticide residues.
17. (E). The conclusion of the argument is that the government should educate the public about the dangers of
inactivity and poor diet in order to stop the spread of type-2 diabetes. The basis for the claim is that inactivity and poor
diet are the main factors in developing type-2 diabetes. You are asked to find a choice that will show that this plan
likely will not work.
(A) The fact that schools educate middle school students about a disease that is generally "acquired in adulthood" does
not address the effectiveness of an adult education plan sponsored by the government.
(B) The fact that the public already has access to this information through the Internet, does not say anything
predictive about the effectiveness of the plan. Even with access to the information, there is a good chance that most
people are not exposed to the information.
(C) Just because food companies encourage the public to indulge in unhealthful snacks, does not mean that a program
that teaches them to do otherwise would not be successful.
(D) The fact that the government has not set aside money for such a program, does not say much about the projected
program's effectiveness.
(E) CORRECT. The choice states that healthful foods and exercise programs are beyond the financial means of
many people. This suggests that even with the best planning, the program might not achieve its goals simply because
people cannot afford to follow the program's advice.
18. (D). The argument focuses on the relative cost of goods and services in Countries X and Y due to an exchange rate
that has historically favored the currency of Country Y. The argument presents an apparent discrepancy: the citizens of
Country Y often take their vacations in Country X, yet rarely purchase clothing or electronics in Country X, despite
the fact that those items are more expensive in their home country, even when sales taxes are taken into account. The
correct answer will be one that indicates some reason why it might not actually be cheaper for citizens of Country Y
to purchase clothing and electronics in Country X.
(A) The fashion preferences of the citizens of Country Y do not directly explain their buying habits. To be relevant,
this statement would have to go further to link fashion preferences to purchasing decisions: for example, it would have
to continue …and are therefore willing to pay more for the goods available in their own country.
(B) The fact that stores in Country Y receive the latest fashions and technology earlier than stores in Country X does
not address the buying habits of the citizens. Even if you could assume that the citizens of Country Y demand the latest
goods as soon as they are available, it still would not directly explain their buying habits. To be relevant, this statement
would have to go further to link consumer preferences to purchasing decisions: for example, it would have to continue
…and the citizens of Country Y are willing to pay more in their own country just to acquire the latest goods as
soon as they are available.
(C) The attitude of the citizens of Country X is irrelevant to an argument about the purchasing habits of the citizens of
Country Y.
(D) CORRECT. This choice states that the government of Country Y imposes tariffs on imported goods. This
suggests that perhaps items that are purchased in Country X and brought into Country Y become prohibitively
expensive because of the tariffs and could explain the spending habits of the citizens of Country Y.
(E) The currency of Country Z is irrelevant to why citizens of Country Y rarely purchase clothing or electronics in
Country X.
19. (A). The basic structure of this argument is fact that mold is almost always found in places where there is
substantial moisture, so therefore, to avoid mold and the resultant mold poisoning, then people should take steps to
prevent wet areas. This argument assumes that wet areas occur first, causing mold to grow. Conversely, this
assumption requires that the mold growth itself does not occur first, creating wet areas as a result.
(A) CORRECT. The argument depends on the assumption that the reason mold and wetness are observed together is
that wet areas cause mold growth. If the reverse causation (mold causes wetness) were true, then keeping all plumbing
in good condition to prevent leakage would do little to prevent the growth of mold. This choice eliminates the
alternate causation.
(B) If most homeowners knew enough about plumbing to determine whether theirs is in good condition, then the
recommendation made in this argument would be more useful. However, this is not an assumption on which the
argument depends.
(C) Even if mold could grow in dry areas, the fact that mold is almost always found in wet areas is still valid. This is
the fact upon which the argument is based, so the argument does not depend on the unnecessarily absolute assertion
that mold cannot grow in dry areas.
(D) Even if some varieties of mold are harmless, the conclusion of this argument, that one should make sure to keep
all internal plumbing in good condition to prevent leakage and minimize mold growth, could still be valid.
Therefore, this argument does not depend on the unnecessarily absolute assertion that no varieties of mold are
harmless.
(E) Whether mold spores can be filtered from the air may be relevant to a conclusion about the health effects of mold
in the home, but it is not directly relevant to this conclusion, that one should make sure to keep all internal
plumbing in good condition to prevent leakage and minimize mold growth.
20. (D). This argument discusses a plan with the stated goal "to prevent overcrowding." Two points of view are
represented in the argument: one is that of the town zoning board, the originators of the plan, and the other is that of
critics of the plan. The question asks for information that would most support the claims of the critics of the plan.
Conversely, the correct answer would most weaken the logic of the town zoning board.
(A) How other towns fared under similar zoning plans is irrelevant to this argument, unless additional information
were presented to connect the situation of this town to those. You have no such connecting information, so this
choice neither strengthens nor weakens either point of view.
(B) How long it has been since the construction of the last school in town is irrelevant to this argument. The argument
addresses methods to prevent overcrowding, an issue that is not directly related to school construction.
(C) The argument never mentions property taxes, so how property taxes in this town compare to those in neighboring
towns is irrelevant.
(D) CORRECT. The argument states that the town zoning board limited the number of new buildings that can be
constructed in the town in any given year. The goal of this plan is to prevent overcrowding, but only does so
indirectly: the town zoning board plan limits the number of new buildings, not the number of new town residents.
Either an apartment building or a single family house would only count as one new building, yet the apartment building
could draw many more new residents to the town than would the single family house. This plan might not be the best
way to achieve the stated goal, and thus this statement supports the claims of the critics of the plan.
(E) The distance to the nearest garbage dump is irrelevant to an argument about how to prevent overcrowding.
21. (C). The conclusion of the argument is that the retail price of chocolate is certain to increase within six months.
This claim is based on the fact that the wholesale price of cocoa solids and cocoa butter has increased significantly
and is likely to stay high. The argument assumes that the retail price of chocolate is driven by the wholesale price of
cocoa, and ignores the cost of the other ingredients in chocolate. To evaluate the conclusion of the argument, it is
necessary to examine whether this assumption is valid.
(A) The price of cocoa solids and cocoa butter during other periods of poor harvest is irrelevant. There is no guarantee
that the market for chocolate would respond the same way in this case.
(B) The willingness of consumers to spend more for chocolate is irrelevant when evaluating whether chocolate will be
more expensive due to a shortage of cocoa beans.
(C) CORRECT. If the price of other ingredients in the chocolate has dropped, then the decrease could offset the
higher price of cocoa, and the retail price of chocolate could remain steady.
(D) The percentage of cacao trees affected by the fungus is irrelevant. Regardless of the percentage, it would not
change the fact stated by the argument that there was an unusually meager harvest of cocoa beans this year.
(E) It may seem useful to determine whether the fungus can be eliminated within the next six months. However, the
conclusion was not about the fungus or the cacao crop, but
21. (C). The conclusion of the argument is that the retail price of chocolate is certain to increase within six months.
This claim is based on the fact that the wholesale price of cocoa solids and cocoa butter has increased significantly
and is likely to stay high. The argument assumes that the retail price of chocolate is driven by the wholesale price of
cocoa, and ignores the cost of the other ingredients in chocolate. To evaluate the conclusion of the argument, it is
necessary to examine whether this assumption is valid.
(A) The price of cocoa solids and cocoa butter during other periods of poor harvest is irrelevant. There is no guarantee
that the market for chocolate would respond the same way in this case.
(B) The willingness of consumers to spend more for chocolate is irrelevant when evaluating whether chocolate will be
more expensive due to a shortage of cocoa beans.
(C) CORRECT. If the price of other ingredients in the chocolate has dropped, then the decrease could offset the
higher price of cocoa, and the retail price of chocolate could remain steady.
(D) The percentage of cacao trees affected by the fungus is irrelevant. Regardless of the percentage, it would not
change the fact stated by the argument that there was an unusually meager harvest of cocoa beans this year.
(E) It may seem useful to determine whether the fungus can be eliminated within the next six months. However, the
conclusion was not about the fungus or the cacao crop, but rather the retail price of chocolate. The time frame for the
elimination of the fungus would only be relevant to the short-term retail price of chocolate if you make several
additional assumptions: that the harvest immediately increases, and that the resulting glut of cocoa beans immediately
offsets the unusually meager harvest of cocoa beans this year, and that the wholesale price of cocoa immediately
drops to its pre-fungus level, and that the retail price would not be affected in the meantime. You cannot make all of
these assumptions, so the answer to this question is irrelevant to the conclusion
.
22. (A). The head of engineering has concluded that the one-time doubling of costs for the raw material caused the
steady decline, over two years, of profit margins. The engineer's conclusion rests on the assumption that there is a
connection between the one-time raw material price increase and the two-year steady decline in profit margins.
Alternatively, something else could have caused either a steady two-year decline in revenues or a steady two-year
increase in costs; if so, this event is more likely to be the cause of a steady decline in profit margins over the same
two-year period. As a result, this would weaken the engineer's conclusion that finding a new source for the raw
material will improve profit margins.
(A) CORRECT. New competitors have caused a steady two-year decline in revenues. This weakens the engineer's
contention that the one-time doubling of costs for the raw material is the cause of the steady two-year decline in
profit margins.
(B) The fact that the region's mines are producing less than they did before the earthquake does not indicate anything
about the cost of the raw material; it would be necessary to show that cost is not reduced in order to weaken the
engineer's conclusion.
(C) The amount of raw material produced by other regions does not indicate anything about the cost of the raw
material; it would be necessary to show that cost is not reduced in order to weaken the engineer's conclusion.
(D) The use of a different raw material does not indicate anything about the cost of that raw material; it would be
necessary to show that cost is not reduced in order to weaken the engineer's conclusion.
(E) Although mining the raw material may become more cost-effective for the mine (that is, cheaper), this does not
tell you what will happen to the price they charge for the material when selling to the company producing the product
in question. It would be necessary to show that the company's cost is not reduced in order to weaken the engineer's
conclusion.
23. (E). The argument concludes that children are more likely to attend college if they are sent to private high schools
instead of public high schools. The basis for this claim is the higher percentage of graduates of private schools
pursuing college education. It is assumed that public schools are inferior to private schools as a training ground for
college. Any statement that provides an alternate explanation for the fact that public school graduates attend college at
lower rates than private school graduates would weaken the argument. Another way to interpret this question involves
the concepts of correlation and causation. The argument's premise states that private school attendance (versus public
school attendance) is highly correlated with college attendance. The conclusion of the argument is essentially that
private school attendance CAUSES college attendance (and therefore, parents ought to send their children to private
schools to ensure eventual college attendance). This conclusion depends on the assumption that the causation does
NOT work the other way — in other words, that "readiness or desire to attend college" does not influence the choice
of public or private school. Any evidence that readiness or desire to attend college DOES influence the choice of
public or private school will weaken the argument.
(A) While higher test scores might increase students' chances of admission to college, they are unrelated to whether
students will actually attend college. Even if one could prove that earning higher test scores makes a student more
likely to attend college, this statement would not weaken the argument, but rather strengthen it.
(B) Since the conclusion centers on the likelihood of attending college, economic and financial considerations are
outside the scope of the argument.
(C) Since the amount of need-based aid is not directly related to whether a student will attend college, this statement
is outside the scope of the argument.
(D) While better athletic opportunities could increase students' chances of admission to college, they are unrelated to
whether students will actually attend college. Also, even though the advantages of public school mentioned in this
statement were taken into account by the study, the proportion of graduates of public schools attending colleges
remains substantially lower than the proportion of graduates of private schools.
(E) CORRECT. This answer choice demonstrates that the difference in the percentage of graduates attending
colleges stems not from any advantage provided by private schools but from the fact that a subset of the graduates of
public high schools simply choose to pursue a different career path. In other words, 30% of the graduates of public
schools voluntarily choose not to pursue a college education. Yet 65% out of the 70% of graduates remaining end up
in colleges. This statement indicates extremely high college matriculation rates for students who want to attend
college after graduation from public high schools. Using the concepts of correlation and causation, this answer choice
provides the alternative causation for the correlation observed. In other words, "desire to attend college" is already
lower in the rural areas where public schools happen to be located. According to this evidence, attendance at private or
public school is the effect, not the cause, of "desire to attend college."
24. (A). Only two pieces of information are given about Airline A's standing room "seats" proposal. First, that it is
geared toward increasing revenue in order to counteract declining profits. And second, that, since the proposal relates
to passenger safety, it must be approved by the Federal Aviation Administration. Airline A must have concluded that
the cost of implementation of its proposal is less than the revenue that the new seats will generate.
(A) CORRECT. Since Airline A knows that its proposal would have to comply with safety standards, it must have
concluded that the cost of compliance is worth it. In other words, the only way for Airline A to achieve its goal of
increasing profit is to implement ideas that will generate more revenue than they cost. Airline A must therefore have
concluded that the standing room only "seats" meet this criteria.
(B) The statements in the passage imply nothing about whether Airline A believes that the Federal Aviation
Administration will approve the proposal. Although Airline A must believe that the proposal has a chance of being
approved (otherwise it's unlikely to have proposed it), the airline might have proposed its specific plan knowing that it
might not be approved or, that it might have to be changed in certain ways.
(C) Airline A's goal is simply to "counteract declining profits" caused by the high cost of jet fuel. This does not mean,
however, that the proposal must fully mitigate the cost of jet fuel. As long as the proposal increases revenue without a
corollary increase in cost, it will in some way (even if it's relatively small) counteract declining profits.
(D) The passage does not mention any other ways that Airline A has considered increasing revenue. Therefore, it is
impossible to conclude anything about Airline A's perception of its standing room "seats" proposal to any other ideas.
(E) The statements in the passage do not address Airline A's view regarding the safety of the standing room only
"seats." It is very possible that Airline A views its proposal as safe and sees no conflict between passenger safety and
increasing revenue, much less that it has made any determination about the relative importance of these two issues.
25. (D). The letter writer believes that if criminal penalties for drug use are eliminated, the incidence of armed
robbery and other violent crimes will decrease. In support of that belief, the letter writer offers the second boldface
portion of the argument: an assertion that crimes are committed by drug users because they need money to buy
expensive illegal drugs, and that if drugs were legal and therefore cheaper, the crimes would become unnecessary. The
first boldface portion of the argument mentions an observed relationship between drug use and other crimes: when
drug use declines, other crimes decline as well. This observation is counter to the letter writer's ultimate claim.
(A) The letter writer forecasts that violent crime will decline even if drug use is decriminalized. The first boldface
portion does not offer support for that forecast, but rather evidence that violent crime decreases when anti-drug laws
are enforced. The second boldface portion is not the letter writer's forecast, but rather the support given for it.
(B) The first boldface portion is an observation that violent crime decreases when anti-drug laws are enforced; that
observation weighs against the letter writer's main position, but falls short of refuting his claim that violent crime will
decrease as a result of decriminalizing drug use. The second boldface portion is support for the letter writer's main
position, not the position itself.
(C) The first boldface portion shows a direct relationship between a decline in drug activity and a decline in violent
crime, but the letter writer does not argue that future events are predicted by this relationship. In fact, the letter writer
ultimately claims the opposite: that violent crime will decrease when criminal penalties for drug use are eliminated,
even if drug use increases as a result.
(D) CORRECT. The first boldface portion shows a direct relationship between a decline in drug activity and a decline
in violent crime, but the letter writer claims that violent crime will decrease when criminal penalties for drug use are
eliminated, even if drug use increases as a result. If true, the information in the second boldface section explains why
the letter writer makes the claim that the high cost of illegal drugs is the reason drug users commit violent crimes, so
cheaper, legalized drugs will cause crime to decline.
(E) The first boldface portion is presented by the letter writer as true. However, the second boldface is not an
inference drawn from the first boldface portion; rather, it contradicts the first boldface portion.
26. (B). Although the premises of this argument suggest only a correlation between smoking and anxiety or
nervousness, the argument has a causal conclusion: it concludes that smoking causes individuals to be anxious and
nervous (i.e., that A causes B). Any assumption in a causal argument must support the causal "direction" of the
conclusion, that A causes B as opposed to some other explanation. Often, assumptions support a causal conclusion
either by eliminating an alternate cause for the conclusion (that C did not cause B) or by demonstrating that the
causation, if one exists, is in the proper direction (that B did not cause A).
(A) The argument concludes that smoking causes anxiety and nervousness. Whether these maladies lead to more
serious health problems is not relevant to the conclusion.
(B) CORRECT. For smoking to be the cause of anxiety and nervousness (A caused B) it must be true that these
individuals were not more likely to be anxious and nervous before they started smoking. If smokers had these
preconditions, which contributed to their decision to begin smoking (B caused A), your conclusion — that smoking
causes these maladies — would be incorrect.
(C) The argument concludes that smoking causes anxiety and nervousness. The number of survey respondents is not
relevant to the conclusion.
(D) The argument concludes that smoking causes anxiety and nervousness. The awareness of the health problems
related to smoking is not relevant to the conclusion.
(E) The argument is not based on the immediate impact that smoking has on anxiety and nervousness. Moreover, the
argument never compares some smokers to other smokers.
27. (D). Because of the speculation that the tuberculosis outbreak in Country X was the result of an outbreak of
tuberculosis in Country Y, health officials in Country X have proposed requiring all visitors from Country Y to
undergo a medical examination. You are asked to find a choice that suggests that this proposal will not have the
desired effect of curbing the spread of tuberculosis in Country X.
(A) This has no bearing on the situation between Country X and Country Y.
(B) This suggests only that the proposal would not prevent ALL cases. But even if the proposal does not prevent all
cases, it could help prevent many.
(C) This suggests only that the proposal would not catch ALL carriers of the disease from Country Y. But even if the
proposal does not prevent all cases, it could help prevent many.
(D) CORRECT. This suggests that the visitors from Country Y are not the source of the disease. Thus, testing them
would likely do little to curb the spread of the disease.
(E) This does not suggest that the proposal will not help curb the spread of the disease. If the visitors from Country Y
are indeed carriers, then their refusal to visit Country X will help curb the disease.
28. (D). The argument concerns the economic impact on restaurants in Prohibitionland if the service of alcoholic
beverages is banned. It presents evidence that, despite restrictions on the service of alcohol in certain areas of
Prohibitionland, sales taxes in restaurants in those areas rose at a higher rate than for those in other parts of
Prohibitionland, suggesting that the ban would not have any adverse economic impact. You are asked to support the
restaurant proprietors' claim, so the correct answer choice will call the relevance of the seemingly contradictory
evidence into question.
(A). This answer choice may seem to strengthen the argument that banning the service of alcoholic beverages would
have an adverse impact on restaurants. However, as the evidence involves data for the entire year, citing a short-term
negative impact on restaurant visitation at the beginning of the year does not measurably strengthen the argument.
(B) The relative tax rate on food and beverages as compared to other consumer good is irrelevant here.
(C) A gradual decline in alcohol consumption over the past 20 years would suggest that over time, any ban on alcohol
would have an increasingly small impact on restaurant visitation, weakening the proprietors' argument.
(D) CORRECT. This statement calls the evidence into question by indicating that any measured increase in sales
taxes and, presumably, revenues for restaurants that have been operating under the restrictions last year enacted is
irrelevant, as the restrictions could be argued to be completely different than the total ban that is being proposed. This
answer choice substantially strengthens the proprietors' argument by threatening to make the cited evidence
irrelevant.
(E) The fact that overall sales tax revenue did not increase at a higher rate in the provinces that enacted the restrictions
on alcoholic beverages weakens the proprietors' argument, as it makes the cited evidence more compelling by ruling
out the possibility of different growth rates in the different areas.
29. (A). The passage concludes that the whereabouts of most of the world's most valuable artwork is probably
unknown. The basis for this claim is that serious art collectors are discreet when purchasing or selling significant
pieces of art. To weaken this claim, one would need information that demonstrated that serious art collectors, while
discreet in purchasing and selling their artwork, were relatively open with their art once it was in their possession.
Alternatively, one could weaken this claim using information that showed that serious art collectors possessed only a
small fraction of the world's most valuable art. The question asks you to find an answer choice that does NOT weaken
the conclusion.
(A) CORRECT. The fact that the value of a piece of art is subjective is irrelevant to the reasoning of the argument. It
does not present any information that would weaken the link drawn between the whereabouts of valuable artwork and
the fact that serious art collectors are discreet when purchasing or selling such artwork. Thus, this choice does NOT
weaken the conclusion.
(B) The fact that serious art collectors publicize their art shortly after purchasing it means that the whereabouts of
their valuable art must be widely known. This choice makes clear that serious art collectors are discreet only in
purchasing and selling their artwork, and that the whereabouts of newly purchased artwork is unknown, if at all, for
only a short period of time. As such, this choice weakens the conclusion.
(C) If museums own the vast majority of the world's valuable artwork, then the practices of serious individual art
collectors are essentially irrelevant to the location of most of the world's valuable artwork. Therefore, this choice
weakens the conclusion since the public nature of museums means that the whereabouts of most of the world's
valuable work is widely known.
(D) Since the majority of the world's valuable privately held artwork is owned by individuals who are not considered
serious collectors, then the practices of serious art collectors are essentially irrelevant to the location of most of the
world's valuable artwork. This choice weakens the conclusion by removing the link between serious art collectors and
most of the world's valuable artwork.
(E) The fact that the collections of most serious art collectors are often displayed in public settings means that the
whereabouts of their valuable art must be widely known. This choice makes clear that serious art collectors are
discreet only in purchasing and selling their artwork, but that once in their possession, the artwork is typically
unveiled for the public. Therefore, this choice weakens the conclusion.
30. (A). The argument explains that the new "Click It or Ticket" law is generating controversy. Under the new law,
drivers can be cited for not wearing their seat belts, even in the absence of an additional driving infraction. Any
acceptable inference must be directly supported by evidence from the text.
(A) CORRECT. The entire controversy is based on the new law that allows motorists to be cited, even in the absence
of an additional infraction. Thus, it follows that prior to the passage of this law, an additional driving infraction must
have been necessary in order to stop and cite an individual for not wearing a seat belt.
(B) Search and seizure laws are never mentioned in the text. This answer choice is outside the scope of the argument.
(C) Laws in other states are never mentioned in the text. This answer choice is outside the scope of the argument.
(D) Though the text states that the new regulation might save countless additional lives, the effectiveness of the
previous laws are never mentioned.
(E) No preference is stated between law enforcement groups and the citizens' groups. This answer choice is simply an
opinion that is unsubstantiated by the text.
31. (E). The passage provides some specific information about the effects of calorie restriction. In rats and mice, this
diet is known to prolong life by preventing diseases. In a study of moderately overweight humans, insulin levels and
body temperature decreased. A proper GRE inference will be based on the specific information provided, without
relying on any significant assumptions.
(A) The passage states that calorie restriction in mice and rats prolongs life by preventing diseases. The human study
had much more limited findings — that calorie restriction in moderately overweight humans decreases insulin levels
and body temperature. While these traits are known to be associated with longevity, there are no data that link calorie
restriction itself to prolonged human life. Additionally, calorie restriction may have other unstated effects, unrelated
to longevity. There is no information in the passage that indicates whether these effects are the same in humans as in
mice and rats. Finally, the use of the term humans is far too general; the study dealt only with moderately overweight
humans and so any inference would need to be restricted to this subset of individuals.
(B) While the passage indicates that certain traits known to be associated with longevity are found in moderately
overweight humans who reduce their calorie intake, this is far removed from the conclusion that calorie intake will
actually increase a human's lifespan. Additionally, as with answer choice (A), the use of the term humans is far too
general.
(C) The study observed that individuals with the greatest percentage decrease in their calorie intake demonstrated the
greatest decrease in insulin levels and body temperature. This shows a strong correlation between calorie intake and
insulin levels. However, this correlation is not necessarily direct. It is possible that this correlation holds, but only up
to a point. For example, it might be the case that any reduction in calorie intake over 50 percent does not result in any
additional insulin level decreases. Moreover, the passage only draws this correlation for individuals with the greatest
percent decrease in calorie intake. It is very possible that individuals with a relatively low decrease in calorie intake
exhibit the exact same decrease in insulin levels as individuals with a moderate decrease in calorie intake.
(D) The study makes no reference to the health of individuals who reduce their calorie intake. It tries to draw some
connection to the longevity of those individuals, but longevity is not the same as health. An individual could live a very
long, unhealthy life.
(E) CORRECT. The passage states that the greatest decrease in insulin levels was observed in individuals with the
greatest percentage change in their calorie intake. This means that some individuals in the study reduced their calorie
intake by a greater percentage than other individuals in the study. The passage also states that the study participants
reduced their individual calorie intakes by at least 25 percent. Thus, one can safely infer that there were some
participants who reduced their calorie intake by more than 25 percent.
32. (D). The argument explains that school officials plan to limit the sugar intake of students in school by replacing
sugary drinks with less sugary drinks in school vending machines. To undermine this plan, the correct answer must
demonstrate that that the consumption of sugar by students will not be dramatically reduced.
(A) A comparison between unsweetened fruit juices and bottled water is not relevant, as these are both less sugary
alternatives to the high calorie sugary drinks.
(B) The intention of the school officials' plan is to limit the intake of sugar by students. Access to sugary drinks
outside of school is certainly a factor in students' total sugar intake. However, the possibility that students might
consume sugar elsewhere fails to undermine the school officials' plan to restrict access to sugary drinks at school. As
a result of the restricted access during school hours, those students who have access to sugary drinks outside of
school may still consume less sugar than they otherwise would.
(C) The fact that certain alternatives to the sugary drinks contain ingredients that cause health concerns is irrelevant to
the argument. Students would still be consuming less sugar.
(D) CORRECT. If most of the sugar that students consume in school comes from snack foods rather than drinks,
replacing the drinks with healthier alternatives will do little to curb the intake of sugar.
(E) This statistic is not directly relevant to the argument, since it is unclear whether these drinks are consumed at
school. If the drinks were consumed at school, this answer choice would strengthen rather than undermine the school
officials' plan.
33. (D). Research indicates that there is a connection between being married and being happy and healthy. Media
commentators have concluded that marriage causes happiness and health. However, one could reasonably conclude
from the research that the cause and effect are the reverse: being happy and healthy makes a person more likely to get
married.
(A) The research compared married people to unmarried people. Neither the researchers nor the media commentators
made any distinction between newlyweds and those who had been married a long time, so this assumption is not
necessary.
(B) The type of wedding is outside the scope of this argument. The research compared married people to unmarried
people, but made no distinction based upon the type of wedding.
(C) At first, this statement may seem necessary—after all, if the commentators conclude that marriage causes
happiness, a lack of depression in married people would certainly support that conclusion. However, the statement is
too extreme. One depressed married person does not invalidate the research indicating that, on average, married
people are healthier and happier than non-married people.
(D) CORRECT. This statement eliminates the alternative interpretation of the research findings—that being happy
and healthy makes a person more likely to get married.
(E) The research compared married people to unmarried people. Neither the researchers nor the media commentators
made any distinction between harmonious marriages and combative marriages, so this assumption is not necessary.
34. (D). The question asks you to analyze the structure of the argument. Specifically, it asks you to determine the
relationship of the two bolded sentences to the argument as a whole. To do this effectively, you need first to
determine the conclusion of the argument. In this case, the conclusion is the final sentence, which follows from the
evidence presented in the rest of the argument. So you know that the correct answer cannot describe either bolded
portion as the author's final conclusion. The first bolded portion is presented as an earlier, erroneous belief. In
evaluating the answer choices, you should look to eliminate any that suggest the author agrees with this first
statement. The second bolded statement shows that the earlier theory — that lactic acid causes soreness — was
incorrect, and thus also provides evidence in support of the author's conclusion. So you know that the correct answer
must describe the second bolded portion as supporting the conclusion. The correct answer must describe both
portions correctly, not just one or the other.
(A) This choice misrepresents the first bolded portion by claiming that the author accepts it as true, when the author
actually presents it as an incorrect, outdated belief. The second portion is also incorrect because it indicates that the
second bolded portion supports the first one.
(B) This choice also misrepresents the first portion, for the reason described above. It corrects the second bolded
portion, however, by describing it as contradictory to the first portion.
(C) The first portion is correctly described as an assertion that the author does not believe to be true. Yet the second
portion is incorrectly described as going against the author's final conclusion.
(D) CORRECT. This choice correctly represents the first portion by saying the author believes it is no longer valid.
The second portion is correctly described as evidence in support of the author's conclusion.
(E) The first portion is correctly described as evidence considered by the author to be invalid. However, this choice
misrepresents the second portion as the conclusion, when it is actually evidence given in support of the conclusion.
35. (B). The conclusion of the argument is contained in the last sentence, that the methods that the prospector had
used to determine the size of the oil deposit must have been inaccurate. The evidence provided is that the
prospector reported a large oil deposit that was later determined to be much smaller in size. You are asked to find an
unstated assumption that makes the conclusion valid based upon this evidence. In order to do this, you need to assume
that there is not another reason why the prospector might have reported a larger oil deposit than actually existed.
(A) It is not necessary to the conclusion that a third party affirmed the company's determination that the oil deposit
turned out to be small. The conclusion accepts that the oil deposit was indeed smaller than indicated by the
prospector, and focuses on the cause of the discrepancy as opposed to the discrepancy itself.
(B) CORRECT. The argument concludes that the prospector's methods resulted in inaccurate measurements of the
size of the oil deposit. This assumes that the prospector did not simply misreport or misrepresent the measurements,
presumably for personal gain. This answer choice addresses the most plausible alternative explanation to the one
given, and is necessary for the conclusion to stand based upon the evidence presented.
(C) The commercial feasibility or profitability of the oil deposit is not integral to the argument, or its conclusion.
This statement is not an assumption that would support the conclusion.
(D) Whether or not the prospector utilized the same methods in regards to measuring the oil deposits in other
locations is not relevant to the argument, or the conclusion.
(E) The fact that the company had a long operating history and experience in drilling oil wells is not relevant, in that
the company's measurements of the size of the oil field are accepted as given in the argument.
36. (C). This passage relates information from two studies concerning high school seniors: the first discusses the
financial responsibilities of high school seniors, while the second explains the coursework in finance taken by typical
high school seniors. On the GRE, a proper response to a draw-a-conclusion question must be directly supported by
evidence from the passage.
(A) Although it might be true that schools would be wise to educate students in finance, this is an opinion; it doesn't
necessarily need to follow from the given evidence. A conclusion must be directly supported by evidence from the
passage without any additional information or assumptions.
(B) The fact that one-third of high school seniors claim significant financial responsibilities to their families does
not necessarily mean that these same students work part-time jobs after school. There are many possible ways that
these students might earn money for their families. If they do work, they might work on weekends or over the
summer, for example.
(C) CORRECT. The first study states that one-third of all high school seniors have significant financial
responsibilities to their families. The second study states that 80% of seniors have opened a bank account, and of this
80%, one-third has bounced a check. The number of seniors that has bounced a check (one-third of 80%) is fewer than
the number of seniors with significant financial responsibilities to their families (one-third of 100%).
(D) The passage states that certain high school seniors who contribute to the food, shelter, or clothing for themselves
or their families rate themselves as having significant financial responsibilities. This does not mean that any high
school senior who contributes to these categories has significant financial responsibilities.
(E) The passage states that one-third of high school seniors say that they have significant financial responsibilities.
This in no way indicates that the other two-thirds have "no" responsibilities. Because no information is given about the
other two-thirds of the students, a reasoned conclusion cannot be drawn about them.
37. (E). According to the statements, the companies that own private aircraft for business use are fully in compliance
with the relevant law, which is summarized. A correct inference will be a statement that must follow from at least part
of the premises given.
(A) It does not have to be true that the law costs the businesses money, as no evidence about the relative costs is given.
(B) This choice is an irrelevant comparison, as the preferences of the executives are not the concern of the
statements.
(C) This choice does not have to follow, as there is no information given about the travel arrangements made by large
companies. The statements only indicate that the majority of private planes are not owned by large companies.
(D) There is no information given about the travel arrangements of upper level executives and no reason to believe that
those with the companies discussed do not comply with their companies' policies.
(E) CORRECT. If, as the statements indicate, the companies are in full compliance with this law, it must be true that
the executives following their guidelines also are.
38. (E). The conclusion of the argument is that insurance companies do not have a significant economic incentive to
delay claim payments to doctors. To weaken this conclusion, an answer choice must provide some significant
economic incentive for insurance companies to be tardy in paying doctors for legitimate medical claims.
(A) While the fact that some doctors who submit accurate bills to insurance companies still receive tardy payments
seems to indicate that there must be something other than errors causing delayed payments, it fails to prove that the
insurance company has an economic incentive to deliberately delay claim payments to doctors. For example, this fact
could simply indicate that the insurance companies are inefficient at handling all of their paperwork.
(B) This choice compares costs insurance companies must absorb due to incorrect bills to costs physicians must
absorb due to tardy payments. However, this information is irrelevant to establishing an economic incentive for
insurance companies to delay claim payments to doctors.
(C) The argument is focused on the payment of legitimate claims; the rising proportion of illegitimate claims does not
establish a clear economic incentive for insurance companies to delay payments of legitimate claims.
(D) The types of billing errors made by doctors' offices does not establish any economic motive for insurance
companies to make a practice of delaying payments to doctors.
(E) CORRECT. This choice articulates a logical chain that establishes a clear economic motive for insurance
companies to be tardy in paying doctors for legitimate medical claims. If insurance companies delay payments to
doctors, this results in a 10 percent increase in overhead costs for physicians. These costs ultimately result in higher
fees that doctors charge to insurance companies. Insurance companies, in turn, raise the premiums they charge
consumers for health coverage. This choice states that the insurance companies increase their fees to consumers far
more than the doctors increase their fees to insurance companies, enabling the insurance companies to pocket the
difference; therein lies the economic motive for insurance companies to be tardy in paying doctors for legitimate
medical claims.
39. (D). Farmers in developing countries claim that the global price of wheat is low because American farmers
produce too much of the grain. They also claim that American farmers produce too much wheat because they have no
incentive to manage their crops, since the US government will buy whatever wheat American farmers cannot sell on
the open market. You are asked to find a choice that weakens the claims of the farmers in developing countries that
removing the American subsidy would cause the price of wheat to rise.
(A) That there are uses for wheat that is not eaten is irrelevant here. This does not address any aspect of the farmers'
claims.
(B) The fact that buyers of wheat can predict their needs in advance is irrelevant here, because the text indicates that
American farmers do not pay attention to actual demand for wheat.
(C) In this argument, the global market for soybeans is irrelevant to the global market for wheat, which is a different
commodity with different demand, supply, and pricing structures.
(D) CORRECT. The farmers assume that the sole cause of the wheat surplus is the United States. This answer choice
suggests that other countries would modify their output to counterbalance any reduction on the part of the United
States, keeping prices constant instead of allowing them to rise.
(E) The price of another crop is largely irrelevant. Moreover, the fact that the price of sorghum, a non-subsidized
crop, is lower tends to support, rather than weaken, the claims of the farmers.
40. (E). The researchers claim that Delta-32 prevents its carriers from contracting the plague. They support this claim
by noting that a strikingly large percentage of descendants of plague survivors carry the mutation. You are asked to
find an assumption underlying the claim.
(A) The argument is specific to the relationship between Delta-32 and resistance to the plague. Other diseases are
irrelevant.
(B) Again, the argument is specific to the relationship between Delta-32 and resistance to the plague. Other diseases
are irrelevant.
(C) Delta-32 may have existed in its current form before the 16th century and the merit of the argument would not
change.
(D) The argument does not claim that Delta-32 prevents all bacteria-caused disease.
(E) CORRECT. The researchers claim that Delta-32 prevented its carriers from contracting the plague on the basis
of its presence in descendants of plague survivors. But it is theoretically possible that these descendants carry the
mutation Delta-32 because the plague mutated the genes of their ancestors. In order to claim that the mutation
prevented the plague, you must assume that the plague did not cause the mutation Delta-32.
41. (E). Consider the structure of the library funding argument. First, a proposal is made (the legislature asks the
public to approve a bond to fund libraries with money outside the existing budget). Second, the argument
acknowledges that the decision maker (the public) supports the intended outcome of the proposed project (building
and improving public libraries). Third, the argument urges rejection of the proposal anyway (since the money could
come from a better source — money in the existing budget that is currently being wasted). Finally, the argument
implies that the decision maker should reject this proposal because it can achieve the intended outcome by some
better means.
(A) In the library funding argument, the decision maker (the public) actually wants the outcome of the proposal
(libraries) but wants to achieve that outcome via a different path. Here, in contrast, parents are the decision makers,
but the proposed outcome (dessert first) is desired only by the proposal maker (children). Further, the proposed
solution (dessert only if dinner is finished) is not an alternate way to achieve the original proposal (dessert first).
(B) This argument is unlike the library funding argument because it focuses on the negative consequences of a
proposal and its effects on a third party (neighbors without alarm systems).
(C) In this argument, a proposal is made by terrorists: they kidnap victims in order to gain publicity. The decision
maker is this country. Unlike the library funding argument, here the decision maker does not agree with the proposed
outcome (increasing publicity for the terrorist group).
(D) In this argument, an employer proposes a specific pay increase. The decision maker is the labor union, and here
the proposal offers the union only one part of what it wanted. In the library argument, the decision maker is urged to
reject a proposal with a good outcome in order to achieve that outcome by other means. Here, the decision maker is
urged to reject the proposal because it is incomplete.
(E) CORRECT. This argument follows the same format as the library funding argument. First, a proposal is made (the
student wants to stay up late to finish his homework). Second, the argument acknowledges that the decision maker (his
parents) supports the intended outcome of the proposed project (doing well in school). Third, the argument urges
rejection of the proposal anyway (since the student wastes time earlier in the evening that could be spent on school
work). Finally, the argument implies that the decision makers should reject this proposal because they can achieve the
intended outcome by some better means.
42. (C). You are asked to determine what you can infer based upon the given information; on the GRE, whatever you
infer must be based only upon the argument itself and cannot extend beyond the scope of that argument.
(A) The argument says nothing about requirements in other states; this choice is out of scope.
(B) While it may be illegal for those under 21 to consume alcohol, the argument makes no mention of special rules
for this age category. Indeed, the argument says that hunters have to sign the pledge, not just hunters aged 21 and over.
(C) CORRECT. The argument says that the hunter must have completed a safety program within the past five years. If
he has not, then he will have to do so before he can be eligible for a permit. (Note that this choice doesn't say he will
get a permit if he completes the program again; you don't know what other requirements he may not meet. You only
know that he will have to retake the program, at least, before he can become eligible.)
(D) This choice mentions someone who isn't 18 years old. Someone who isn't 18 can be either younger than 18 or
older than 18 and, according to the argument, those older than 18 are eligible for permits if they complete all of the
requirements. according to the argument, those older than 18 are eligible for permits if they complete all of the
requirements.
(E) While this may be true as a general rule, it is outside of the scope of the argument, which mentions nothing about
danger or why these rules were enacted. In addition, any requirements cannot ensure that nobody gets hurt; they can
only help to minimize risks.
43. (A). The argument claims that wide dissemination of wireless access is now a practical way to meet urban needs,
based on the evidence of its successful use in rural areas. The author then must assume that urban areas provide no
additional problems for wireless use.
(A) CORRECT. This choice confirms an assumption of the argument and thus strengthens the conclusion.
(B) This choice weakens the argument because it damages the assumption that urban areas pose no extra problems for
wireless use.
(C) This choice is irrelevant because it provides information about another rural area; the conclusion concerns urban
areas.
(D) This choice weakens the argument because it damages the assumption that urban areas pose no extra problems for
wireless use.
(E) This choice is an irrelevant distinction. The argument mentioned all three groups as in need of this service. The
suggestion that one group needs it more than the others is irrelevant to the conclusion.
44. (D). The conclusion of this argument is that many weight loss efforts fail because people eat more calories than
they intend to consume. The first boldface portion is a factual premise ("Studies have shown…") that there is an
observed correlation between keeping a diet record and losing weight successfully. This premise (indirectly) supports
the researchers' conclusion. The second boldface portion is another supporting premise, this one citing a specific
study showing that dieters who do not keep a diet record eat far more than they realize.
(A) The first boldface is not the conclusion, it is an observed fact. The second boldface is evidence that the
researchers' conclusion is correct, but is not evidence that the first boldface is correct.
(B) The first boldface is a fact that supports the researchers' theory, but it does not explain why their conclusion is
correct—the other premises do so.
(C) The first boldface is a fact that supports the researchers' theory, but it does not illustrate the truth of that theory—
the second boldface does. The second boldface is a fact that supports the researchers' theory; it is not a competing
theory.
(D) CORRECT. The first boldface (diet record = diet success) is a basis for the researchers' conclusion that many
weight loss efforts fail because people consume more than they intended. The second boldface directly illustrates
how weight loss efforts of a certain group failed for exactly that reason.
(E) The first boldface is a factual statement, not a theory. Furthermore, the first boldface supports the theory of the
researchers; it is not something they have disproved.
45. (B). This argument concludes that the city should convene a conference of relevant parties to create opportunities
for out-of-work young people. The argument's premise is that the retirement of the baby boomers will create
shortages. The argument assumes the efficacy of its conclusion — in other words, that the conference will actually be
effective in creating job opportunities. Attacking an assumption is an effective way to weaken an argument.
(A) If anything, this choice strengthens the argument. If immigration does not provide a labor pool, it is more likely
that a shortage will ensue.
(B) CORRECT. The argument assumes that it is feasible to affect employment patterns by government
encouragement and/or action. If that assumption is denied, the conclusion is weakened, as the conference would be
pointless.
(C) This choice makes an irrelevant distinction. It doesn't matter if the best positions require skills, as long as the
majority are available to the unskilled unemployed in question.
(D) Knowing that a small proportion of baby boomers will not retire on schedule does not significantly weaken the
argument. The argument relies on general estimates, not on exact numbers.
(E) If anything, this choice strengthens the argument. If these people are unaware of these opportunities, it would be
positive to convene to plan how to reach them.
46. (A). The argument itself provides one example of the Doppler effect: a siren decreases in perceived pitch as it
approaches, passes and then moves away from an observer. The answer choice that most closely describes a similar
scenario is a correct illustration of the Doppler effect.
(A) CORRECT. The passengers on the westbound train see the eastbound train approaching, passing, then traveling
away from them. The sound of the horn, therefore, will decrease in pitch for the westbound passengers.
(B) The passengers on the westbound train see the eastbound train approaching, passing, then traveling away from
them. The sound of the horn, therefore, will decrease in pitch for the westbound passengers. This choice, however,
says that the sound will increase in pitch.
(C) Relative to the passengers on the eastbound train, the eastbound train and its horn are in fact standing still! The
Doppler effect only describes a perceived change in pitch that occurs when the source of a sound is in motion
relative to the observer, a situation that is not illustrated by this choice.
(D) Relative to the passengers on the eastbound train, the eastbound train and its horn are in fact standing still! The
Doppler effect only describes a perceived change in pitch that occurs when the source of a sound is in motion
relative to the observer, a situation that is not illustrated by this choice.
(E) Relative to the passengers on the eastbound train, the eastbound train and its horn are in fact standing still! The
Doppler effect only describes a perceived change in pitch that occurs when the source of a sound is in motion
relative to the observer, so while it is true that the eastbound passengers would hear their train's horn at its true pitch,
this situation does not illustrate the Doppler effect.
47. (E). The author concludes that one will only be able to determine the age of a Brazilian ash by counting its rings if
the temperature in the tree's environment never exceeds 95 degrees Fahrenheit. The author bases this conclusion on
the fact that the tree loses rings when the temperature exceeds that level. However, if the number of rings lost by a
Brazilian ash at high temperatures can be predicted, it may be possible to determine the age of a tree even if the
temperature exceeds 95 degrees.
(A) The argument says nothing about precipitation. This answer choice is out of scope since it would require a number
of other assumptions to make it relevant to the argument's conclusion.
(B) Whether other trees share this feature is irrelevant; the argument focuses only on the Brazilian ash.
(C) The number of days of excessive heat needed to cause the tree to lose rings is irrelevant.
(D) The thickness of the rings is irrelevant.
(E) CORRECT. The conclusion is that the rings will be a reliable measure only if the temperature never exceeds 95
degrees. This is true only if there is no way to predict how many rings would be lost when the temperature does
exceed 95 degrees. If it were possible to predict this, one might be able to assess the age of a tree using its rings even
if the temperature had exceeded 95 degrees.
48. (E). The text tells you that celiac disease results when the body mistakes gluten for a harmful pathogen, causing
damage to the intestine. You are also told that gluten is a protein found in certain grains, and that people suffering
from celiac disease must eliminate it from their diets. Finally, you are told that symptoms of the disease include
cramps, bloating, and anemia. You need to find an answer choice that is inferable from these facts alone.
(A) Anemia is just one of several symptoms of the disease. You do not know whether everyone who has the disease
will also develop anemia.
(B) You do not know whether eliminating gluten will cure the disease, only that people with the disease must not eat
gluten. Perhaps the disease will exist anyway in a latent form.
(C) You do not know whether the symptoms mentioned are also symptoms of other conditions.
(D) You do not know whether gluten is found only in grains. It may exist in other foods as well.
(E) CORRECT. If the body mistakes gluten for a harmful pathogen, then it must be true that the body cannot always
recognize harmless substances.
49. (E). This argument proposes that culture does not influence the process by which the mind distinguishes colored
objects. In support of the conclusion, the argument notes that all languages with six colors name the same colors, as
well as all languages with three colors. To weaken the conclusion, you need evidence to support the counter-argument
that culture does influence how the mind distinguishes colored objects. Notice the logical difference between
distinguishing color names and distinguishing colored objects. For instance, a speaker of English can visually
distinguish two objects with different shades of red, even if he or she cannot easily name the difference. Some of the
wrong answers attempt to confuse these two processes.
(A) Irrelevant. It may be interesting to observe that no language has more than eleven basic color terms. However, this
observation neither weakens nor strengthens the argument that culture influences how the mind perceives color
variations.
(B) Irrelevant. In fact, this statement may slightly strengthen the argument: if every language permits speakers to
describe subtle color variations, then it might be argued that human color perception is independent of language, since
language would then not be "boxing in" the speaker.
(C) Irrelevant. The term red may encompass both red and yellow, but that doesn't mean that speakers of the language
can't see the difference between red and yellow. In the same way, the English word blue covers many shades that
English speakers with normal vision can distinguish (sky blue, royal blue, etc.)
(D) Strengthens. If speakers of languages without a blue-green distinction refer to the sky or tree leaves to clarify
their meaning, then they obviously see a difference between the sky and tree leaves. Using the sky or tree leaves as
reference points is no different from using fruit names for colors (e.g., orange, peach). This evidence reinforces the
idea that humans have a common basis for perceiving colors.
(E) CORRECT. If Tarahumara speakers are less able to identify differences between blue and green objects than
Spanish speakers, then it can be argued that the lack of a blue-green distinction in the Tarahumara language influences
how Tarahumara speakers actually perceive colors. It should be noted that this evidence does not completely prove the
point — the causality could actually work the other way (e.g., Tarahumarans could share a genetic defect that causes
blue-green color blindness, and so they never developed a distinction in their language). Also, if the experiment is not
well designed, the difficulty in identification could simply come from the lack of a handy term to capture the
difference of blue and green. However, the evidence as given does support the hypothesis that culture influences color
perception.
50. (A). This question is an Analyze the Argument Structure question; the best approach is to identify the conclusion,
and then determine how each boldface portion of the argument relates to the conclusion. The conclusion of this
argument appears in the third sentence: doctors determined that Anopheles mosquitoes were responsible for
transmitting the disease to humans. The first boldface statement tells you that malaria was named based upon the
original, but incorrect, view that the disease was caused by "bad air"; the statement does not support the conclusion.
The second boldface statement provides the specific mechanism by which the mosquito is responsible for human
infection; it therefore supports the conclusion. More simply, the first statement does not support the conclusion,
while the second statement does.
(A) CORRECT. The first statement does follow from the original, but incorrect, view that the disease was caused by
"bad air." It weakens the conclusion by supporting an incorrect conclusion. The second statement is a supporting
premise which explicates (or explains) why the conclusion is true.
(B) A conjecture is a hypothesis or a supposition, but the first statement is not a conjecture; it is a fact. The second
statement contradicts the original conclusion, but it does not contradict the fact provided in the first statement.
(C) To illuminate a thing is to clarify or explain it. The first statement does not explain any assumption (or the original
conclusion, for that matter); it merely states a fact that follows from the original, erroneous conclusion. The second
statement does not confirm anything in the first half of the argument; rather, it supports the later, correct conclusion.
(D) The first statement did not cause the erroneous conclusion; rather, it is a result of that conclusion. The second
statement is a premise that supports the correct conclusion.
(E) The first statement does provide additional detail about the original, erroneous conclusion. The second statement
is not the actual conclusion (or judgment); it is a premise in support of the later conclusion.
51. (A). In this argument, a cause-and-effect relationship is presented between American scientists signing long-term
contracts with foreign companies and the government's restrictions on stem cell research. This cause-and-effect
relationship is the key to the correct answer.
(A) CORRECT. If American scientists signed the contracts because of US restrictions, you can infer that the new
companies they signed with were under fewer restrictions. Therefore, at least some foreign companies must work
under fewer restrictions than some American companies do.
(B) While it is possible that once the restrictions are banned, American companies will want to hire more scientists
and will seek them overseas, there are too many unknowns between the premises you have have been given and this
conclusion. It is doubtful that an increase in the number of immigrating stem cell research scientists would have a
significant impact on the number of foreign professionals overall.
(C) This passage is about government restrictions; you are given no information about financial backing. Beware of
extreme statements such as in all parts of in all parts of the world.
(D) You are not given any information regarding America's current or future position in terms of stem cell research.
Though restrictions and scientists switching companies are two issues related to a company's prosperity, you are
given no information about how these directly affect America's position.
(E) You are not given any information that will help you predict the behavior of the scientists in the future.
52. (D). In the argument, the critics make the error of mistaking correlation with causation. While it is true that most
employees were reassigned to the mission to Mars, it is not established that the mission to Europa was scrapped in
order to move the employees to a different project, or if there was some other motivation that caused the demise of
the Europa project. The conclusion that NASA presents is that the critics, in saying that public interest is the key
motivator behind NASA's actions, are misinformed.
(A) Irrelevant. The conclusion is based on the critics' opinions on causation, and this answer fails to address the issue
of what motivated NASA in its decision-making process. Beware of reading too deeply into the information presented.
The fact that the percentage of spending is going to go down could indicate many possible scenarios, including a)
NASA is unhappy with the progress of the project and plans to cut future spending or b) they expect the development
of the shuttle to be completed by 2013.
(B) Weakens. If public interest determines its budget, NASA has strong motivation to keep public interest high.
Additionally, this is a highly unlikely answer because it concerns "budget," and "budget" is not mentioned in the
original argument.
(C) Irrelevant. This statement differentiates between the opinions of some scientists and the opinions of others, but
sheds no light on the motivations behind NASA's decisions. Not only is the answer choice only indirectly related to
your statement, it also adds very little new information.
(D) CORRECT. This answer choice provides an alternate reason why NASA scrapped its plan for sending an
unmanned vessel to Europa — the Tokyo telescope provides the information NASA would have attained from the
mission, making the mission unnecessary.
(E) Irrelevant. The conclusion deals with NASA's motivations; this statement is about the motivations of American
citizens. There is no direct relationship between what Americans see as the future of space exploration and the
motivations behind NASA's decision-making.
53. (B). The argument claims that no one with a compromised immune system should travel to Nicaragua's "Mosquito
Coast," due to the risk of contracting dengue fever or malaria. The correct answer choice is the choice that least
strengthens the argument. Therefore, the wrong answers must each clearly strengthen the argument in one of several
ways: 1) increasing the probability of infection, the primary negative outcome; 2) increasing the harmful effects of
infection; or 3) increasing the harmful effects of alternative outcomes — for instance, demonstrating the negative
side effects of prevention methods.
(A) Strengthens. Since the method of preventing or treating one of the diseases can have bad side effects, the danger is
made worse — particularly for people with compromised immune systems.
(B) CORRECT. If the government were to succeed in this effort, there would be somewhat less risk to travelers with
compromised immune systems. This would weaken the conclusion that such travelers should avoid the Mosquito
Coast. If the effort were to fail, it would produce no change in the danger to such travelers. Either way, this answer
choice does nothing to strengthen the argument that such travelers should avoid the Mosquito Coast.
(C) Strengthens. Since a compromised immune system doesn't effectively fight either dengue fever or malaria, people
with such immune systems are at increased risk of infection by such diseases.
(D) Strengthens. The populations of both mosquitoes grow during rainy season; thus, the chance of contracting either
disease also grows.
(E) Strengthens. The insect repellent may be the most effective available, but it can also have a terrible side effect.
Using this repellent is one of the options for dealing with the risk of mosquito-borne disease. Since this option is
more dangerous than before, the travel advisory is strengthened.
54. (D). According to the text, the Peruvian government claims that Inca Trail treasures would have deteriorated
without a new permit program that has restricted the number of tourists. Supporting statements would likely
emphasize one of the following two ideas. First, A high number of tourists causes the deterioration of Inca Trail
archaeological sites, and so a reduced number reduces the deterioration. (Note that this causal connection, while
reasonable, cannot be assumed. Evidence that would support this connection might include a comparison with similar
ruins that did not benefit from a permit program and that have experienced greater deterioration, or a comparison
between two periods of time (before and after 2001) for Inca Trail ruins.) Second, other results of the permit
program (e.g., new revenue) help prevent archaeological damage along the Inca Trail.
(A) Irrelevant. The increase in pay may have resulted from the permit program, and it may be reasonable to assume that
this increase in pay has led to greater satisfaction in the job and hence, perhaps, to greater care for historical artifacts.
However, this chain of reasoning is too speculative to strengthen the argument appreciably.
(B) Irrelevant. Local villages may have seen a drop in income as a result of the restrictions on tourist numbers, but this
does not strengthen or weaken the claim that the permit program prevented Inca Trail ruins from deteriorating.
(C) Irrelevant. The funds are a positive result of the permit program, but if these funds are used to protect or preserve
archaeology elsewhere, then they do not impact the preservation of ruins specifically on the Inca Trail.
(D) CORRECT. The more rapid deterioration of similar ruins elsewhere supports the claim that that the permit
program has helped prevent deterioration of Inca Trail ruins. Notice that this evidence does not rise to the level of
absolute proof; other differences between the ruins might explain the different rates of deterioration. However, this
evidence clearly supports the hypothesis that the permit program was successful.
(E) Irrelevant. Without the permit program, it is possible (though far from certain) that the number of tourists hiking
the Inca Trail would have risen together with the total number of tourists in Peru. However, an increase in the number
of tourists on the Inca Trail would not necessarily have led to greater deterioration of archaeological treasures on the
trail.
55. (E). The chairman claims that same-day spending at downtown businesses by people attending performances at the
Farmsley Center has contributed to the economic revitalization of downtown Metropolis. His argument depends on it
being true that this spending represents an increased flow of money into the economy of downtown Metropolis. If, for
example, the $83 per visitor that he cites is money that would have been spent in downtown businesses even if the
Farmsley Center had not been built, the chairman's argument would be unsound. Therefore, you should look for an
answer choice that suggests that the $83 per visitor is indeed money that probably would not have been spent in
downtown Metropolis in the absence of the Farmsley Center.
(A) Irrelevant. You do not know what contributions to the city the business group has in mind. Perhaps the chairman is
being honored for activities unrelated to the Farmsley Center, or perhaps he is being honored simply because
Metropolis residents feel civic pride at having a grand performance space downtown. You need more information
before you can say that this prize constitutes evidence that the Farmsley Center has helped to revitalize the downtown
economy.
(B) Irrelevant. Expensive restaurants may be a sign of the economic revitalization of downtown Metropolis, but they
do not tell you what causal factors led to that revitalization.
(C) Irrelevant. In the absence of information specifically relating the architecture of the Farmsley Center to spending
at downtown businesses, you cannot say that the architect's international standing has helped in the economic
revitalization downtown.
(D) Irrelevant. The Farmsley Center may be hosting performances that would otherwise have taken place at other
downtown venues, but this does not mean that extra money is being spent downtown.
(E) CORRECT. If suburbanites are coming to Metropolis to see performances at the Farmsley Center, they are
bringing money from out of town and spending it in downtown Metropolis. This inflow of money supports the idea
that the Farmsley Center has contributed to the economic revitalization of downtown Metropolis. Notice, by the way,
that this information about suburbanites by no means constitutes an iron-clad proof of the chairman's contention. (It is
conceivable, for instance, that in the absence of the Farmsley Center the suburbanites would have found other reasons
to visit Metropolis.) Since this is a Strengthen the Conclusion question, however, you do not need to find an answer
choice that proves the conclusion — just one that makes the conclusion more likely.
56. (D). The argument presents two retirement account options. In the first, taxes are paid when money is withdrawn
upon retirement; in the second, taxes are paid when the money is initially deposited into the account. The author
assumes that it is better to pay taxes on the contributions than on the withdrawals when the total contribution amount
is smaller than the total available for withdrawal. But the amount paid in taxes, whether on contributions to the fund or
on withdrawals from the fund, depends on both the amount of money contributed or withdrawn and on the tax rates
applied to those contributions or withdrawals. Thus, to evaluate the argument, it would be helpful for you to know the
amounts of money involved as well as the relevant tax rates.
(A) It would be useful to know both how much is contributed and the value of the account upon retirement. Knowing
just the first figure, however, doesn't allow the worker to figure out which retirement account option would permit
him to pay less in taxes.
(B) It might be useful if you knew when and how the tax rates would increase, but "in the future" is too vague. The tax
rates may increase while the worker is still employed, or they may not increase until the worker has retired. Without
knowing, you cannot evaluate the conclusion any further.
(C) This may tell you how wisely the worker invested the funds, but it does not provide any information about the tax
consequences, which are the focus of the conclusion.
(D) CORRECT. If a worker knows the relative tax brackets to expect during the different stages, he or she can better
predict whether it would be less expensive to pay taxes on the contributions or on the withdrawals.
(E) It would be useful to know both how much is contributed and the value of the account upon retirement. Knowing
just the second figure, however, doesn't allow the worker to figure out which retirement account option would permit
him to pay less in taxes.
57. (C). The argument presents a discrepancy between the percentage of survey respondents who reported that they
did vote in an election (71%) and the percentage of eligible voters who did vote in that election (60%). An explanation
of the discrepancy will often hinge upon explaining why the apparent conflict does not apply, distinguishing the
situation from others, or providing an additional premise that resolves the discrepancy.
(A) If the margin of error is ± 5%, this is the same as saying that the 71% figure could be as low as 66% (or as high as
76%). This accounts for less than half of the discrepancy between 71% and 60%.
(B) This choice does not address the stated discrepancy between those who said they voted and those who actually did
vote. You have not been told that voters are required to maintain a primary residence in the home country at the time
of the election, nor have you been told that voters must be physically present in the home country in order to cast a
vote.
(C) CORRECT. One explanation for the discrepancy between these two results is the possibility that people who do
vote will respond to surveys at a higher rate than people who do not vote — in other words, people who do vote are
overrepresented in the survey's results. This is an additional premise that would explain the higher percentage of
individuals polled indicating that they voted.
(D) While this may be true, the poll did not ask people if they intended to vote; rather, it asked people if they had
already voted in a past election.
(E) While this may account for some percentage of the discrepancy, you are not given any numerical data to indicate
that the entire discrepancy can be explained; "some" could mean only 2 people. In addition, the confused people
wouldn't necessarily have responded that they did vote (when thinking about a different election) even though they
didn't vote in the national election. They could just as easily have reported that they did not vote (when thinking about
a different election) even though they did vote in the national election.
58. (B). You are presented with a paragraph of premises and asked to resolve the problem they present: how to
minimize the spread of flu via air travel. The correct answer will need to rely specifically on the premises and will not
require you to make any inferences or assumptions.
(A) The passage states that the infection can be spread by coughing. The flu virus, therefore, can reach the other
passengers in the "closely-packed environment" before it enters any filters that might kill the virus.
(B) CORRECT. Vaccines provide significant protection against developing the virus (not 100% protection, but you
are asked to "minimize" the impact of air travel, not eliminate it entirely). If all passengers are vaccinated against the
virus, many of those who otherwise would have developed the disease will not, and, therefore, won't spread it to
others.
(C) Anyone can contract the virus and subsequently spread it; the mentioned populations are merely "especially
vulnerable" to it. Infected people traveling to another place can infect children, senior citizens, and others who have
stayed in their home regions.
(D) The passage states that the infection can be spread by coughing; while it may be true that the virus can also spread
via hand contact, this information is not stated in the passage.
(E) The passage states that people who develop symptoms before travel begins likely would not make the trip; weeding
out those with observable symptoms, then, won't "minimize" the role of air travel because there aren't that many
people in this category. The larger danger is those who may be infected but have not yet developed symptoms.
59. (B). The journalist cites data about the success of tall people, then concludes that employers have an unconscious
bias in favor of tall people. The journalist is assuming that employer bias is the only explanation for the data; look for
an answer choice that throws this explanation into question.
(A) Irrelevant. Gender comparisons are irrelevant in interpreting the journalist's data on CEOs, since those data are
only about male CEOs. Likewise, gender comparisons are irrelevant in interpreting the journalist's data about the
general population, since the passage says those data have been corrected for the influence of gender and age.
(B) CORRECT. If socioeconomic status is correlated to both height and educational attainment, you would expect
taller people to be, on average, better educated. The economic success of tall people could then be attributed to their
higher levels of educational attainment rather than to employer bias.
(C) Irrelevant or Weakens. Professional basketball players, with their above-average height and above-average pay, do
account for a small part of the correlation between height and pay. And insofar as being tall is useful in the game of
basketball, the high wages of tall players can be explained without reference to any unconscious bias on the part of
their employers. Thus, the case of professional basketball players may weaken the journalist's argument slightly.
However, basketball pros constitute a tiny fraction of the labor force, so their high pay accounts for an extremely
small portion of the $789-per-inch wage advantage conferred by height. Also, basketball players tend to be very tall,
so their high wages cannot explain wage differentials among short people — why, for instance, people who are 4'11"
tend to earn more than do people who are 4'10". Finally, the high wages of basketball players do nothing to undermine
the journalist's interpretation of the data on male CEOs, because basketball players are not CEOs.
(D) Irrelevant. An HR professional might unconsciously favor tall people (or good-looking people, or charismatic
people, etc.) without herself being tall (or good-looking, or charismatic, etc.).
(E) Irrelevant. Unless you are told why a tall person tends to stay in a particular job less long than does a short person,
you cannot say whether this information bolsters or undermines the journalist's argument.
60. (D). The argument is concerned with how public school teachers are compensated. It suggests that educational
experts believe that a system of teacher compensation based on performance rather than seniority would help to retain
exceptional teachers and maintain quality instruction. The correct answer is the one that most undermines this
contention of the educational experts.
(A) Irrelevant. The fact that many factors contribute to job satisfaction and teaching performance neither weakens nor
strengthens the argument for a performance-based pay structure for public school teachers.
(B) Irrelevant. Nothing in the argument indicates that one universal system of compensation must be adopted. It is very
possible that several effective models of performance-based pay could be developed and implemented successfully.
(C) Strengthens. This choice indicates that many young teachers are extremely frustrated by the traditional pay
structure, in which financial advancement is directly tied to seniority. Thus, these teachers would likely welcome a
change that allows them more rapid opportunity for financial advancement.
(D) CORRECT. Weaken. This choice indicates that collaboration among teachers is integral to high-quality
instruction and that a system of compensation based on teacher performance reduces collaboration. Thus, the effect
of a merit-based system of pay would be to undermine quality instruction, which is one of the two stated goals of the
educational experts.
(E) Strengthens. The educational experts' argument in favor of performance-based compensation is bolstered if
standardized tests scores have dramatically risen in school districts that have instituted such pay structures.
No comments:
Post a Comment